USMLE Full Length Exam 2

Approved & Edited by ProProfs Editorial Team
The editorial team at ProProfs Quizzes consists of a select group of subject experts, trivia writers, and quiz masters who have authored over 10,000 quizzes taken by more than 100 million users. This team includes our in-house seasoned quiz moderators and subject matter experts. Our editorial experts, spread across the world, are rigorously trained using our comprehensive guidelines to ensure that you receive the highest quality quizzes.
Learn about Our Editorial Process
| By Chachelly
C
Chachelly
Community Contributor
Quizzes Created: 513 | Total Attempts: 592,897
Questions: 48 | Attempts: 220

SettingsSettingsSettings
USMLE Full Length Exam 2 - Quiz

Questions and Answers
  • 1. 

    An oncologist recently discovered that certain cancerous cells secrete a protein named ca- 1panc. Using this protein, he developed a new blood test to detect this type of cancer. He performed the blood test on 1,000 patients. One hundred of these patients had the cancer, and the test came back positive for 60 of them, while for the remaining 40 patients the test was negative. Nine hundred of the patients did not have the cancer; however, the test was positive for 100 of them. In the remaining 800, the test came back negative. Which of the following numbers represents how well the test identifi ed those who had the cancer?

    • A.

      10.0%

    • B.

      37.5%

    • C.

      60.0%

    • D.

      88.8%

    • E.

      90.0%

    • F.

      95.2%

    Correct Answer
    C. 60.0%
    Explanation
    1. The correct answer is C. It is important to understand that the question is asking for the sensitivity, the proportion of people who have the disease and test positive out of all the people who have the disease. It is calculated by TP / (TP + FN), where TP means true-positive and FN means false-negative. The true-positives in the vignette represent those with the cancer who correctly tested positive with this new test (n = 60). False-negatives are those with the cancer who tested negative with the new test (n = 40); thus, 60 / (60 + 40) = 60%. Screening tests theoretically would aim to identify all those with the disease, and therefore high sensitivities are desired. In this case 60% represents a low number, and the ca-1panc blood test would not be a good screening test for the cancer.
    Answer A is incorrect. The 10.0% figure represents the prevalence of the disease, calculated as total cancer/total people (100/1,000).
    Answer B is incorrect. The 37.5% figure is the positive predictive value, or the probability that someone with a positive test (ca-1panc) truly does have the cancer. The predictive values vary with how prevalent a disease is in the population. It is calculated as TP / (TP + FP) (where TP means true-positive and FP means false-positive) or 60 / (60 + 100) = 37.5%.
    Answer D is incorrect. The 88.8% figure represents the specificity of the blood test, which measures the proportion of the people who don’t have the disease and test negative out of all the people who don’t have the disease. It is important to correctly detect those without the disease in order to prevent them from undergoing unnecessary treatment or studies that could be painful or harmful to the patient. It is calculated as TN / (TN + FP) (where TN means true-negative and FP means false-positive), or 800 / (800 + 100) = 88.8%.
    Answer E is incorrect. The 90.0% figure simply represents the percentage of people without the cancer, 900/1000.
    Answer F is incorrect. The 95.2% figure is the negative predictive value, or the probability that the person with a negative test really does not have the cancer. It is calculated as TN / (TN + FN) (where TN means true-negative and FN means false-negative), or 800 / (800 + 40) = 95.2%.

    Rate this question:

  • 2. 

    A 27-year-old woman who is pregnant at 32 weeks’ gestation presents to the emergency department following a motor vehicle accident. Results of fetal heart monitoring are reassuring, and there is no evidence of rupture of membranes. Radiologic studies show a fractured femur. The patient is admitted to the hospital for expectant management and is placed on strict bed rest with delivery planned at 37 weeks’ gestation. Which of the following medications would be most appropriate for preventing deep venous thrombosis in this patient?

    • A.

      Heparin

    • B.

      Indomethacin

    • C.

      Prostaglandin E2

    • D.

      Streptokinase

    • E.

      Warfarin

    Correct Answer
    A. Heparin
    Explanation
    2. The correct answer is A. Pregnancy is considered to be a state of hypercoagulability with an increased risk for deep venous thrombosis (DVT) and pulmonary embolus. A major concern is that a DVT can lodge within the pulmonary arterial system. This, in turn, can result in pulmonary hypertension, hypoxia, and, in the worst case, right-sided heart failure and death. Pregnant women who have an indication for anticoagulation therapy (in this case, both stasis and endothelial injury secondary to her trauma) should be treated with an anticoagulant. Heparin is the preferred anticoagulant because it does not cross the placenta.
    Answer B is incorrect. Indomethacin is a nonsteroidal anti-inflammatory agent and is contraindicated in the third trimester of pregnancy because it will close the ductus arteriosus prematurely. Indomethacin is also used as a tocolytic in preterm labor.
    Answer C is incorrect. Prostaglandin E2 can be applied topically to ripen the cervix and promote induction of labor; however, in this patient, immediate induction of labor is not desired.
    Answer D is incorrect. Streptokinase is a thrombolytic agent used in the treatment of early myocardial infarction.
    Answer E is incorrect. Warfarin cannot be used because it can cross the placenta and has been implicated in nasal hypoplasia and skeletal abnormalities in the fetus in the first trimester. It also causes diffuse central nervous system abnormalities, particularly optic atrophy during pregnancy.

    Rate this question:

  • 3. 

    A 38-year-old woman with a history of type 2 diabetes mellitus gives birth to a term male infant. Immediately after birth, the infant is noted to be cyanotic and tachypneic. His hypoxemia quickly worsens over minutes, and he is taken to cardiac catheterization, where a balloon is guided to perforate the atrial septum. He is also given an infusion of prostaglandin E1. The infant’s hypoxia stabilizes, and he is later taken for defi nitive, corrective surgery. Which of the following is the underlying pathophysiology of this infant’s hypoxemia?

    • A.

      Coarctation of the aorta

    • B.

      Concomitant ventricular septal defect

    • C.

      Delayed closure of the ductus arteriosus

    • D.

      Failure of the aorticopulmonary septum to spiral

    • E.

      Overriding aorta

    Correct Answer
    D. Failure of the aorticopulmonary septum to spiral
    Explanation
    3. The correct answer is D. The neonate suffers from transposition of the great vessels, in which the aorta rises from the right ventricle and the pulmonary artery from the left ventricle. It is a common congenital heart defect in the children of mothers with diabetes. Without a shunt, transposition is incompatible with life. If the infant is born without the shunt, an artificial shunt can be created by balloon atrial septostomy, and prostaglandin E1 can be given to salvage whatever patent ductus arteriosus might remain. Once stabilized, the infant can then be taken for corrective surgery. Transposition is also associated with maternal diabetes.
    Answer A is incorrect. Coarctation of the aorta would not cause early cyanosis and is not associated with transposition of the great vessels.
    Answer B is incorrect. A concomitant ventricular septal defect (VSD), while sometimes present in patients with transposition of the great vessels, is not the underlying pathophysiology of this infant’s cyanosis. Additionally, if this patient had a VSD, there would have been little reason to create an additional shunt in the atria.
    Answer C is incorrect. A prostaglandin E1 infusion delays the closure of the patent ductus arteriosus; however, this confers a protective advantage in patients with transposition of the great vessels, as it allows for mixing of blood. It is not the underlying pathophysiology of transposition of the great vessels.
    Answer E is incorrect. An overriding aorta is one component of tetralogy of Fallot in addition to ventricular septal defect, right ventricular outflow tract obstruction, and right ventricular hypertrophy. Although tetralogy of Fallot is a cause of early cyanosis, an isolated overriding aorta would not cause the emergent hypoxia seen in this infant.

    Rate this question:

  • 4. 

    A 34-year-old man presents to his primary care physician with night sweats, a fever of 38º C (100.2º F), and weight loss of 5 kg (12 lb) over the last 3 months. A CT scan demonstrates mediastinal lymphadenopathy, and results of a biopsy of the node are shown in the image. Which of the following drugs is part of the multidrug regimen that would be used to manage this patient’s disease?

    • A.

      Cyclosporine

    • B.

      Hydroxyurea

    • C.

      Imatinib

    • D.

      Isoniazid

    • E.

      Vinblastine

    Correct Answer
    E. Vinblastine
    Explanation
    4. The correct answer is E. The patient presents with the classic signs of Hodgkin’s lymphoma, and the biopsy demonstrates the presence of Reed–Sternberg cells with reactive lymphocytes. Reed–Sternberg cells are large with lobed nuclei that look like “owl’s eyes” and appear to be two halves mirroring each other. Vinblastine is part of the ABVD regimen (Adriamycin, Bleomycin, Vinblastine, Dacarbazine) regimen used to treat Hodgkin’s lymphoma. Vinblastine works by inhibiting microtubule formation necessary to build the mitotic spindle, so that cells in mitosis get stuck in metaphase. It is used to treat many solid tumors as well as Hodgkin’s and non-Hodgkin’s lymphoma. Adverse effects include alopecia, constipation, myelosuppression, and (rarely) neurotoxicity.
    Answer A is incorrect. Cyclosporine is an immunosuppressant used in transplant patients and autoimmune disorders, but would not be appropriate for a patient with Hodgkin’s lymphoma. It works by inhibiting the production and release of interleukin-2, which is necessary to induce cytotoxic T lymphocytes. Adverse effects include gastrointestinal upset, headache, and tremor but amazingly little myelosuppression.
    Answer B is incorrect. Hydroxyurea is used for treatment of sickle cell anemia and to suppress high WBC counts in acute leukemia and chronic myelogenous leukemia. It would not be helpful in this patient, who is presenting with Hodgkin’s lymphoma. Hydroxyurea is an antimetabolite, and although its exact mechanism is unknown, it is believed to work in the synthesis (S) phase of the cell cycle. The major adverse effect to remember is myelosuppression.
    Answer C is incorrect. Imatinib is used to treat chronic myeloid leukemia (CML; a patient would present with increased neutrophils and metamyelocytes) and gastrointestinal stromal tumors, and would not be helpful in a patient with Hodgkin’s lymphoma. Imatinib is a tyrosine kinase inhibitor that specifically inhibits brl-abl in CML, promoting apoptosis in these cells. Adverse effects include weight gain, gastrointestinal distress, musculoskeletal pain, and myelosuppression.
    Answer D is incorrect. Isoniazid is one of the drugs used to treat tuberculosis (other drugs include rifampin, pyrazinamide, streptomycin, or ethambutol). Although the patient presents with some symptoms that would be expected with tuberculosis, such as night sweats, fever, and weight loss, the biopsy results clearly show the Reed–Sternberg cells of Hodgkin’s lymphoma. Isoniazid works by inhibiting mycolic acid synthesis and therefore disrupting the cell wall. Adverse effects include hepatotoxicity, neuropathy, and potentially psychiatric symptoms.

    Rate this question:

  • 5. 

    A 14-year-old high school freshman presents to her family doctor for a sports physical. She has not played organized sports in the past but is in good physical shape. She mentions that she experienced severe leg cramps after trying out for the soccer team last week. The night after the tryouts, she noticed that her urine had a reddish tinge. She has no other medical complaints. Her physician orders an ischemic forearm exercise test, which reveals no increase in venous lactate. Which of the following enzymes is most likely defi cient in this patient?

    • A.

      Cystathionine synthase

    • B.

      Glucose-6-phosphatase

    • C.

      α-1,6-Glucosidase

    • D.

      Glycogen phosphorylase

    • E.

      Lysosomal α−1,4-glucosidase

    Correct Answer
    D. Glycogen phosphorylase
    Explanation
    5. The correct answer is D. This patient suffers from McArdle’s disease, a glycogen storage disorder in which glycogen phosphorylase is deficient in muscle. The enzyme is responsible for liberating individual units of glucose-1-phosphate from branches of a glycogen molecule. Onset of the disease typically occurs in adolescence or early adulthood and is characterized by muscle cramping, rapid fatigue, and poor endurance during exertion. Severe myoglobinuria is also observed in some patients.
    Answer A is incorrect. Homocystinuria is an inborn error of metabolism caused by a defect in cystathionine synthase, the enzyme that converts homocysteine to cystathionine. In addition to Marfan-like features, these patients are at increased risk for a variety of cardiovascular derangements due to increased atherosclerosis, including premature vascular disease and early death.
    Answer B is incorrect. Glucose-6-phosphatase is the enzyme responsible for converting glucose- 6-phosphate to glucose. It is a component of gluconeogenesis. A deficiency of this enzyme causes Von Gierke’s disease, characterized by a severe fasting hypoglycemia, increased glycogen in the liver, hepatomegaly, and increased blood lactate. These findings are inconsistent with the symptoms observed in this patient.
    Answer C is incorrect. α-1,6-Glucosidase is the enzyme responsible for the debranching of glycogen. It is implicated in Cori’s disease, which is a mild form of Von Gierke’s disease with normal blood lactate levels. It is not implicated in McArdle’s disease as it wouldn’t cause the muscle cramping.
    Answer E is incorrect. Lysosomal α-1,4- glucosidase is the defective enzyme in Pompe’s disease, another glycogen storage disorder. The findings in Pompe’s disease typically manifest in early childhood and include respiratory difficulties (due to diaphragmatic weakness), cardiomegaly, and progressive loss of muscle tone leading to early death.

    Rate this question:

  • 6. 

    A 35-year-old man comes to the physician because he has been experiencing bone pain, in addition to confusion, lethargy, recurrent renal stones, duodenal ulcer, and a small nodule on the anterior neck. Laboratory tests show: Calcium: 17 mg/dL Phosphate: 1.0 mg/dL Alkaline phosphatase: 500 U/L Parathyroid hormone: 900 pg/mL Which of the following is most likely to be seen in this patient’s bones?

    • A.

      Hypertrophic osteoarthropathy

    • B.

      Osteitis fibrosa cystica

    • C.

      Osteopetrosis

    • D.

      Osteoporosis

    • E.

      Paget’s disease of the bone

    Correct Answer
    B. Osteitis fibrosa cystica
    Explanation
    6. The correct answer is B. This patient is likely suffering from hyperparathyroidism due to a parathyroid adenoma. Primary hyperparathyroidism causes hypercalcemia, hypophosphatemia, increased alkaline phosphatase activity, and an increase in serum parathyroid hormone (PTH). Hypercalcemia can cause metastatic calcification, including nephrocalcinosis, and development of renal stones and peptic duodenal ulcer disease. Furthermore, PTH elevation can lead to a variety of bone abnormalities, including osteitis fibrosa cystica, a condition that results from excessive bone resorption and fibrous replacement of the marrow, leading to cystic spaces and areas of hemorrhage, or “brown tumors.” Clinically, patients may present with bone pain and fractures.
    Answer A is incorrect. Hypertrophic osteoarthropathy is seen in patients with a variety of illnesses, including lung cancer, sepsis, endocarditis, and inflammatory bowel disease. Patients frequently present with digital clubbing and painful swelling of wrists, fingers, elbows, and other joints. New bone formation is present at the ends of these bones.
    Answer C is incorrect. Osteopetrosis is characterized by brittle, dense, thickened bones that fracture easily. The disease has an autosomal dominant and recessive mode of inheritance
    Answer D is incorrect. Osteoporosis, which is characterized by decreased bone mass, pain, and fractures, is most commonly seen in postmenopausal women and the elderly. Osteoporosis is associated with normal calcium, phosphorus, and alkaline phosphatase levels.
    Answer E is incorrect. Paget’s disease of the bone, like osteitis fibrosa cystica, is a result of excessive bone resorption. In Paget’s disease, the resorbed bone is replaced by a soft, disorganized bone matrix with a mosaic, rather than trabecular, pattern. Alkaline phosphatase activity is increased. Patients can present with bone pain, fractures, deformity, and sensory deficits due to nerve impingement.

    Rate this question:

  • 7. 

    A 45-year-old man comes to the physician with a 3-day history of a temperature of 39° C (102.2° F). He also complains of headache, neck stiffness, and a maculopapular rash on his trunk. A diagnosis of meningitis is made, and a smear and culture of his cerebrospinal fl uid identify Neisseria meningitidis as the causative agent. In the most severe form of meningococcemia, which of the following symptoms is most likely to develop?

    • A.

      Acute renal failure and thrombocytopenia with hemolytic anemia

    • B.

      Fever, migratory polyarthritis, and carditis

    • C.

      Fever, new murmur, small erythematous lesions on the palms, and splinter hemorrhages on the nail bed

    • D.

      Shock, widespread purpura, disseminated intravascular coagulation, and adrenal insuffi ciency

    • E.

      Symmetric ascending muscle weakness beginning in the distal lower extremities

    Correct Answer
    D. Shock, widespread purpura, disseminated intravascular coagulation, and adrenal insuffi ciency
    Explanation
    7. The correct answer is D. Waterhouse-Friderichsen syndrome is a possible complication of meningococcemia. In this disorder, bilateral hemorrhage into the adrenal gland causes adrenal insufficiency. This results in hypotension, tachycardia, a rapidly enlarging petechial skin lesion, disseminated intravascular coagulation, and coma.
    Answer A is incorrect. Hemolytic-uremic syndrome (HUS) is characterized by acute renal failure and thrombocytopenia with hemolytic anemia. HUS can be a complication of infection caused by E. coli O157:H7 and not Neisseria meningitidis.
    Answer B is incorrect. Rheumatic fever is characterized by fever, migratory polyarthritis, and carditis. It may follow group A streptococcal pharyngitis.
    Answer C is incorrect. Fever, a new murmur, Janeway lesions, and nail-bed hemorrhages are all signs of bacterial endocarditis. Acute endocarditis is caused by Staphylococcus aureus and subacute infection can be caused by Streptococcus viridans.
    Answer E is incorrect. Guillain–Barre syndrome is characterized by rapidly progressing ascending paralysis. It is thought to follow a variety of infectious diseases, such as cytomegalovirus, Epstein-Barr virus, HIV, and gastroenteritis caused by Campylobacter jejuni.

    Rate this question:

  • 8. 

    Cyclooxygenase-2 (COX-2) inhibitors, unlike similar nonselective nonsteroidal anti-infl ammatory drugs, may be associated with cardiovascular events, including myocardial infarction. Which of the following proposals might explain why selective COX-2 inhibitors may cause more cardiovascular events than does aspirin? (PGI2 = prostaglandin I2, TxA2 = thromboxane A2)

    • A.

      A

    • B.

      B

    • C.

      C

    • D.

      D

    • E.

      E

    Correct Answer
    E. E
    Explanation
    8. The correct answer is E. PGI2 inhibits platelet aggregation and therefore is an antithrombotic agent. On the other hand, TxA2 increases platelet aggregation and is a prothrombotic agent. COX-2 inhibitors selectively decrease PGI2, leaving the action of TxA2 unopposed. This could well result in increased cerebrovascular and cardiovascular events due to the tonic, unopposed prothrombotic action of TxA2.
    Answer A is incorrect. COX-2 inhibitors do not increase PGI2. They are thought to spare the gastric mucosa because they selectively block the synthesis of other prostaglandins, not because they increase the production of PGI2.
    Answer B is incorrect. COX-2 inhibitors do not increase TxA2. COX-2 inhibitors in general do not increase PGI2 and TxA2, which are downstream products.
    Answer C is incorrect. COX-2 inhibitors and aspirin do not have the same actions.
    Answer D is incorrect. COX-2 inhibitors do decrease PGI2, but aspirin does not increase PGI2 and TxA2. Aspirin is a nonselective COX inhibitor that decreases both PGI2 and TxA2.

    Rate this question:

  • 9. 

    A 30-year-old woman with systemic lupus erythematosus treated with high-dose prednisone comes to her physician with symptoms of anemia. The patient’s blood studies show a low hemoglobin level (10 g/dL), a low serum iron level, an elevated ferritin level, and a low total iron-binding capacity with normocytic RBCs on blood smear. Which of the following is the most appropriate treatment for this patient’s anemia?

    • A.

      Erythropoietin

    • B.

      Ferrous sulfate

    • C.

      Folate

    • D.

      Parenteral vitamin B12

    • E.

      Phlebotomy

    Correct Answer
    A. Erythropoietin
    Explanation
    9. The correct answer is A. This patient’s clinical presentation is consistent with anemia of chronic disease (ACD) in the setting of systemic lupus erythematosus. ACD presents with low serum iron levels, elevated ferritin levels, decreased total iron-binding capacity, and microcytic/ normocytic RBCs on blood smear. ACD resolves if the underlying condition is corrected, but in the absence of a successful primary treatment, erythropoietin can be effective in treating the anemia. Iron therapy is not effective in treating this disorder.
    Answer B is incorrect. Iron therapy is inappropriate in treating anemia of chronic disease, since iron stores are not low.
    Answer C is incorrect. Folate supplementation would be appropriate in macrocytic anemia caused by folate deficiency.
    Answer D is incorrect. Parenteral vitamin B12 therapy is appropriate for pernicious anemia caused by lack of intrinsic factor, which is necessary for vitamin B12 absorption.
    Answer E is incorrect. Phlebotomy is appropriate in treating cases of iron overload, as seen in patients with chronic transfusion therapy and hemochromatosis.

    Rate this question:

  • 10. 

    A 7-year-old girl is brought to the emergency department by her parents because of concerns that she is not growing and not developing appropriately. The parents say that the patient has cold intolerance, easy fatigability, and polyuria. A physical examination is notable for short stature and bilateral papilledema. Thyroid function tests are notable for low levels of triiodothyronine, thyroxine, and thyroid-stimulating hormone (TSH). An MRI shows an enhancing multilobulated suprasellar mass with ring calcifi cation in the region of the sella turcica. If the lesion represents a primary intracranial neoplasm, which of the following is the most likely diagnosis?

    • A.

      Craniopharyngioma

    • B.

      Ependymoma

    • C.

      Hemangioblastoma

    • D.

      Prolactinoma

    • E.

      Thyrotropinoma

    Correct Answer
    A. Craniopharyngioma
    Explanation
    10. The correct answer is A. Craniopharyngiomas account for 80%–90% of neoplasms arising in the pituitary region and are the most common supratentorial tumor of childhood. They originate from squamous rest cells in the remnant of Rathke’s pouch between the adenohypophysis and neurohypophysis. Eighty percent of patients have evidence of endocrine dysfunction at diagnosis; growth hormone deficiency is the most common (75%), followed by gonadotropin deficiency (40%), and ACTH or TSH deficiency (25%). Even though these masses are frequently large at presentation, it is rare for the pituitary stalk to be disrupted; only 20% of patients have prolactinemia, and 10%–15% have diabetes insipidus (DI) secondary to pituitary stalk dysfunction. This patient has symptoms of growth hormone deficiency (short stature), biochemically documented hypothyroidism, DI (polyuria), and increased intracranial pressure as suggested by bilateral papilledema (nonspecific for craniopharyngioma). Imaging in craniopharyngioma may reveal calcification within the tumor.
    Answer B is incorrect. Ependymomas are most commonly found in the fourth ventricle and can result in hydrocephalus; however, it is very rare for ependymomas to cause the hormonal changes evident in this patient.
    Answer C is incorrect. Hemangioblastomas are associated with von Hippel–Lindau syndrome when found with retinoblastomas. Such a tumor would not affect hormonal release.
    Answer D is incorrect. While a prolactinoma can cause many of the same symptoms (pubertal delay/failure) and may present with symptoms similar to those of a craniopharyngioma, it represents a far less likely diagnosis (accounting for 2.7% of childhood tumors) than craniopharyngioma.
    Answer E is incorrect. Thyrotropinomas present with hyperthyroidism without TSH suppression, goiter, visual symptoms, and headache. It would be uncommon for them to suppress growth hormone release, and they are very rare in childhood.

    Rate this question:

  • 11. 

    An obese 56-year-old African-American man with a 25-pack-year history of smoking experiences chest pain associated with an apparent heart attack. The pain radiates to the man’s left shoulder and down his left arm. What is the reason for referred pain to this region?

    • A.

      Lymphatic drainage of mediators of infl ammation and pain

    • B.

      Proximity of sensory nerve fi ber tracts in the anterior horn of the spinal cord

    • C.

      Proximity of sensory nerve fi ber tracts in the posterior horn of the spinal cord

    • D.

      Shared parasympathetic pathways

    • E.

      Shared sympathetic pathways

    Correct Answer
    C. Proximity of sensory nerve fi ber tracts in the posterior horn of the spinal cord
    Explanation
    11. The correct answer is C. Afferent pain fibers of the heart enter the posterior horn of the spinal cord at the same level as the brachial plexus, thus leading to pain that is perceived as being located in the neck and shoulder region.
    Answer A is incorrect. Lymphatic drainage does occur in the left upper quadrant, but it plays no role in the model of referred myocardial pain.
    Answer B is incorrect. Sensory neurons have their origin in the dorsal root ganglion and send their axons to the posterior horn of the spinal cord instead of the anterior horn, which is where efferent neurons arise.
    Answer D is incorrect. The heart and the neck and shoulder region do not share similar parasympathetic innervation patterns.
    Answer E is incorrect. The heart and the neck and shoulder region do not share similar sympathetic innervation patterns.

    Rate this question:

  • 12. 

    A 57-year-old man who is HIV-positive presents to his physician with headache, nausea and vomiting, and a change in mental status. No nuchal rigidity is noted. A lumbar puncture is performed and shows a high opening pressure. A preparation of his bronchoalveolar lavage fl uid with India ink stain is shown in the image. Intravenous treatment is started for the acute condition. Which of the following adverse effects would most likely occur with this patient’s initial treatment?

    • A.

      Arrhythmia

    • B.

      Bone marrow suppression

    • C.

      Gynecomastia

    • D.

      Flushing

    • E.

      Nausea and vomiting

    Correct Answer
    A. Arrhythmia
    Explanation
    12. The correct answer is A. Patients with AIDS are susceptible to a variety of infections that are unusual in the immunocompetent population. Among diseases that cause fever and headache in these patients are Cryptococcus, toxoplasmosis, and central nervous system lymphoma. An encapsulated yeast that stains with India ink is a pathognomonic description of Cryptococcus neoformans, which is a yeast found in pigeon droppings. Infection occurs when patients inhale fungus particles, which can lead to pneumonia. Initial treatment of C. neoformans is intravenous amphotericin B, followed by fluconazole once the patient’s condition is stable. Amphotericin toxicity can cause fever and chills, hypotension, nephrotoxicity, and arrhythmias. The arrhythmias are due to QT prolongation, which is exacerbated by changes in potassium and magnesium levels.
    Answer B is incorrect. Bone marrow suppression is seen with a number of drugs, including flucytosine.
    Answer C is incorrect. Gynecomastia is an adverse effect of fluconazole treatment. The -azole antifungals inhibit ergosterol synthesis. They are used to treat systemic mycoses but are less effective than amphotericin B. Other adverse effects include liver dysfunction and fever.
    Answer D is incorrect. Flushing can be caused by caspofungin, an antifungal medication used to treat aspergillosis infection. Caspofungin inhibits synthesis of an essential component of the fungal cell wall. Other adverse effects include gastrointestinal upset.
    Answer E is incorrect. Nausea and vomiting are seen with flucytosine, which is used to treat systemic fungal infections. Flucytosine inhibits DNA synthesis because it is converted to fluorouracil in vivo.

    Rate this question:

  • 13. 

    A 21-year-old sexually active college student presents to the clinic complaining of odorous green vaginal discharge and itchiness. Multiple oval fl agellated motile organisms are seen on wet mount. She is prescribed an antibiotic treatment. While on therapy she goes to a party, where on consumption of one alcoholic beverage she experiences fl ushing, tachycardia, headaches, and vomiting. The same effects could be observed when alcohol is mixed with which drug?

    • A.

      Ampicillin

    • B.

      Erythromycin

    • C.

      Glipizide

    • D.

      Imipenem

    • E.

      Tolbutamide

    Correct Answer
    E. Tolbutamide
    Explanation
    13. The correct answer is E. The patient consumed alcohol while taking metronidazole for her trichomoniasis. Metronidazole and tolbutamide both have disulfiram-like adverse effects when combined with alcohol use. These effects include flushing, tachycardia, headaches, and vomiting. Tolbutamide is a first-generation sulfonylurea that can also produce these effects.
    Answer A is incorrect. Ampicillin is a penicillin family antibiotic often used to treat gram positive infections or Listeria infections. It does not have disulfiram-like adverse effects.
    Answer B is incorrect. Erythromycin is a macrolide antibiotic and is not commonly used to treat trichomoniasis. It does not have disulfiram-like adverse effects.
    Answer C is incorrect. Glipizide is a second-generation sulfonylurea that does not have the disulfiram-like adverse effects of first-generation sulfonylureas.
    Answer D is incorrect. Imipenem is a carbapenem- type antibiotic with a very broad spectrum of action. The main adverse effect is that it can cause seizures in rare cases. It does not have disulfiram-like adverse effects.

    Rate this question:

  • 14. 

    A woman with a 2-year-old son comes to her physician because she has been unable to conceive a second child for more than a year. The woman is currently breastfeeding her son. Which of the following explains how lactation suppresses ovulation?

    • A.

      Lactation antagonizes estrogen action

    • B.

      Lactation decreases secretion of prolactin

    • C.

      Lactation inhibits secretion of gonadotropin- releasing hormone

    • D.

      Lactation increases secretion of folliclestimulating hormone

    • E.

      Lactation increases secretion of sterility hormones

    Correct Answer
    C. Lactation inhibits secretion of gonadotropin- releasing hormone
    Explanation
    14. The correct answer is C. Lactation is maintained by prolactin secretion from the anterior pituitary. Prolactin prevents ovulation by several mechanisms. It inhibits the secretion of gonadotropin-releasing hormone (GnRH) from the hypothalamus and inhibits the action of GnRH at the anterior pituitary, thus decreasing the secretion of luteinizing hormone (LH) and follicle-stimulating hormone (FSH). Finally, it inhibits the actions of LH and FSH on the ovaries. This woman might have a better chance of becoming pregnant if she were to stop breastfeeding her son.
    Answer A is incorrect. Lactation and the resulting high levels of prolactin do not affect estrogen’s action on the ovary.
    Answer B is incorrect. Lactation is maintained by prolactin. It does not decrease its secretion.
    Answer D is incorrect. Lactation results in decreased secretion of FSH.
    Answer E is incorrect. Lactation does not cause sterility.

    Rate this question:

  • 15. 

    A 5-day-old normally developed boy presents to the emergency department with vomiting and constipation. The mother states that the child has not passed stool since birth but only recently began vomiting. The vomitus has a greenish coloration. On examination, the abdomen is markedly distended and is dull to percussion. Digital rectal examination shows an empty rectum, but stool is passed explosively following the examination. A sweat chloride test is negative. The disorder in this infant arises from cells derived from which of the following embryologic cell populations?

    • A.

      Endoderm

    • B.

      Mesoderm

    • C.

      Neural crest

    • D.

      Neuroectoderm

    • E.

      Surface ectoderm

    Correct Answer
    C. Neural crest
    Explanation
    15. The correct answer is C. The disorder described in the question is Hirschsprung’s disease, a disorder of neural crest cells. Specifically, neural crest cells fail to migrate to distal portions of the colon, leading to a congenital lack of parasympathetic ganglion cells. This produces a functional obstruction of the colon, as peristalsis cannot take place. As a result, the abdomen becomes distended and bilious vomiting eventually results, while the rectum is empty on digital examination. Pathologic exam will reveal a lack of ganglia in Meissner’s and Auerbach’s plexuses and nerve fiber hypertrophy in Meissner’s plexus.
    Answer A is incorrect. The endoderm gives rise to the epithelial lining of much of the gastrointestinal tract, but this is not a disorder of the epithelium.
    Answer B is incorrect. The mesoderm gives rise to smooth muscle, but this patient has Hirschsprung’s disease, a disorder of colonic ganglion cells, not smooth muscle.
    Answer D is incorrect. The neuroectoderm gives rise to most components of the nervous system but not to the ganglia.
    Answer E is incorrect. The surface ectoderm gives rise to the epithelial lining of the lower anal canal, but this patient has Hirschsprung’s disease, which is not a disorder of the epithelium.

    Rate this question:

  • 16. 

    A woman strikes her head in a car crash and is admitted to the hospital, where she begins urinating up to 1 L every few hours and complaining of constant thirst. Which of the following is the most appropriate treatment?

    • A.

      Demeclocycline

    • B.

      Desmopressin

    • C.

      Furosemide

    • D.

      Insulin

    • E.

      Mannitol

    Correct Answer
    B. Desmopressin
    Explanation
    16. The correct answer is B. Desmopressin (dDAVP) is 1-deamino-8-D-arginine vasopressin, an analog of ADH. This woman has central DI caused by trauma to the posterior pituitary. This inhibits secretion of ADH. Repleting her ADH is the most appropriate therapy of the options given. If dDAVP is chosen, the patient’s sodium and fluid status should be carefully monitored, because acute trauma to the posterior pituitary can lead to a triphasic response. In phase 1, the injured pituitary ceases secretion of ADH, resulting in the clinical picture presented in the question stem. In phase 2, the death of pituitary tissue causes the release of ADH stores, leading to fluid retention and hyponatremia consistent with the syndrome of inappropriate ADH secretion. Phase 3 arises from permanent damage to the posterior pituitary, resulting in persistent central diabetes insipidus.
    Answer A is incorrect. Demeclocycline is used to treat the syndrome of inappropriate ADH secretion. This compound acts to inhibit ADH action and would exacerbate her condition.
    Answer C is incorrect. Furosemide is a loop diuretic and is likely to exacerbate her condition.
    Answer D is incorrect. Insulin is an inappropriate treatment. Central DI shares only the symptoms of polydipsia and polyuria with diabetes mellitus. The treatments and causes are completely different.
    Answer E is incorrect. Mannitol is an osmotic diuretic that would exacerbate her condition.

    Rate this question:

  • 17. 

    A 65-year-old man presents to his family doctor for a regularly scheduled check-up. He is recently widowed and appears unkempt at this visit. His past medical history is signifi cant for alcoholism, hypertension, and type 2 diabetes mellitus. His current medications are hydrochlorothiazide, metformin, and clonidine. On physical examination, his blood pressure is 158/90 mm Hg. His body mass index is 33 kg/ m2, and he reports a 4.5-kg (10-lb) weight loss over the past 3 months. On further questioning the patient admits to owning a gun and states that he has thought repeatedly of shooting himself in the head. Which of the following actions is most appropriate in the care of this patient?

    • A.

      Hospitalize voluntarily or involuntarily

    • B.

      Prescribe an antidepressant

    • C.

      Refer to a psychiatrist

    • D.

      Schedule for a regular visit

    • E.

      Tell the patient’s children so that they can intervene

    Correct Answer
    A. Hospitalize voluntarily or involuntarily
    Explanation
    17. The correct answer is A. This patient has a number of risk factors for suicide, including an organized plan, male gender, older age, depression, ethanol abuse, and single status. Other risk factors include loss of rational thinking, lack of social support, and chronic illness. If a patient has suicidal ideation and is deemed at high risk to himself, hospitalization is an appropriate intervention.
    Answer B is incorrect. This patient may be given an antidepressant while in the hospital, but a prescription alone is insufficient care for this patient. In addition, antidepressants may take up to 1 month to have a therapeutic effect.
    Answer C is incorrect. While the patient will certainly benefit from referral to a psychiatrist, he should be hospitalized because he represents an imminent threat to himself.
    Answer D is incorrect. Scheduling this patient for a regular visit will ignore his symptoms of major depressive disorder and place him at risk for harming himself.
    Answer E is incorrect. The patient has not been declared incompetent; it is therefore a breach of confidentiality to tell his children without his consent.

    Rate this question:

  • 18. 

    A 5-year-old girl is brought to the emergency department with acute onset of projectile vomiting and severe headache. Her parents report that over the past couple of months her gait has become increasingly unstable. Her medical history is negative for seizures and signs of meningitis. Physical examination is notable for truncal ataxia and papilledema. CT reveals a mass at the cerebellar vermis. Which of the following is most likely to be seen on histological examination of tissue from this mass?

    • A.

      Cells with round, regular, centrally located nuclei surrounded by a perinuclear halo

    • B.

      Prolifi c vasculature and receding rows of nuclei

    • C.

      Rod-shaped perinuclear inclusions

    • D.

      Round calcifi cations

    • E.

      Small cells with high nuclear:cytoplasmic ratios surrounding the vasculature

    Correct Answer
    E. Small cells with high nuclear:cytoplasmic ratios surrounding the vasculature
    Explanation
    18. The correct answer is E. Gait disturbances and ataxia are results of the tumor impinging on the cerebellar vermis. Some of the symptoms arise from obstructive hydrocephalus. Medulloblastomas, ependymomas, and hemangioblastomas are childhood primary brain tumors that can result in hydrocephalus by obstruction of the fourth ventricle. However, the patient’s gait disturbances and truncal ataxia indicate medulloblastoma, the most common pediatric brain tumor, as the most likely cause. Medulloblastomas are a form of primitive neuroectodermal tumor and appear as small blue cells arranged in perivascular rosettes. They are most often found at the cerebellar vermis.
    Answer A is incorrect. “Fried egg” cells have round, regular nuclei with a perinuclear halo and are typical of oligodendrogliomas, which are more common in adults.
    Answer B is incorrect. Pseudopalisading tumor cells and microvascular proliferation indicate glioblastoma multiforme, which is more common in adults.
    Answer C is incorrect. Rod-shaped blepharoblasts (basal ciliary bodies) are typical of ependymomas.
    Answer D is incorrect. Psammoma bodies, which are round, extracellular concretions of calcium, are typical of meningiomas, which are not likely to cause the symptoms listed. In addition, meningiomas are more common in adults.

    Rate this question:

  • 19. 

    A patient presents to the emergency department with a crushing injury to her right ankle. A medical student working there is told to evaluate the vascular integrity of the patient’s right lower extremity. She feels for femoral, dorsalis pedis, and posterior tibial pulses and fi nds that they are intact and symmetric bilaterally. In which of the following locations did the student palpate the posterior tibial pulse?

    • A.

      Between the two heads of the gastrocnemius muscle

    • B.

      Deep in the popliteal fossa

    • C.

      Immediately anterior to the medial malleolus

    • D.

      Immediately posterior to the lateral malleolus

    • E.

      Immediately posterior to the medial malleolus

    • F.

      On the dorsal surface of the foot

    Correct Answer
    E. Immediately posterior to the medial malleolus
    Explanation
    19. The correct answer is E. Recall that blood supply to the entire leg comes from the femoral artery. The femoral artery runs anteriorly on the thigh until reaching the adductor hiatus, where it dives deep and becomes the popliteal artery. The posterior tibial artery branches from the popliteal artery and runs deep in the calf until the ankle, where it lies in the superficial fossa immediately posterior to the medial malleolus. When there is a penetrating or crushing injury to the leg, it is important to palpate this artery and assess for symmetry.
    Answer A is incorrect. The posterior tibial artery does run between the heads of the gastrocnemius muscle, but it is deep in the leg and is not easily palpable. You should remember from anatomy that the palpable pulses in the leg are the femoral artery in the groin, the popliteal artery in the popliteal fossa on the posterior aspect of the knee, the posterior tibial posterior to the medial malleolus, and the dorsalis pedis in the first interosseous space on the dorsum of the foot.
    Answer B is incorrect. The popliteal artery is found in the popliteal fossa on the posterior aspect of the knee.
    Answer C is incorrect. There is no palpable artery anterior to the medial malleolus.
    Answer D is incorrect. The popliteal artery gives off three branches in the lower leg, one for each compartment: the posterior tibial, the fibular, and the anterior tibial artery. To assess the anterior tibial artery, one can palpate the dorsalis pedis pulse on the dorsal surface of the foot. The branches of the anterior tibial artery that supply the lateral ankle are too small to palpate.
    Answer F is incorrect. The dorsalis pedis artery runs along the dorsal surface of the foot and is a continuation of the anterior tibial artery.

    Rate this question:

  • 20. 

    A 53-year-old man with a long-standing history of allergic rhinitis and asthma presents with uveitis, mild hearing loss, numbness and tingling in his right hand, and diffuse joint pain for the past 10 days. Physical examination shows weak to absent left knee patellar refl exes (right knee refl ex strong and intact). Laboratory studies show a markedly elevated eosinophil count. A diagnosis is made, and the patient is treated with cyclophosphamide. Further laboratory studies show elevated serum levels of the most common autoantibody associated with this condition. What structure is primarily targeted by the autoantibodies that are most likely elevated in this patient’s serum?

    • A.

      Acetylcholine receptors

    • B.

      Neutrophils

    • C.

      Oligodendrocytes

    • D.

      RBCs

    • E.

      TSH receptors

    Correct Answer
    B. Neutrophils
    Explanation
    20. The correct answer is B. This patient has Churg–Strauss syndrome (also known as allergic granulomatosis and angiitis), which is one of a trio of diseases (Wegener’s granulomatosis and microscopic polyangiitis being the others) that are commonly referred to as the ANCA (antineutrophil cytoplasmic antibody)-associated vas culitides (i.e., diseases causing inflammation of blood or lymphatic vessels). Fifty to seventy percent of patients have elevated levels of ANCA, usually the perinuclear pattern of staining type. Patients often have preexisting asthma and allergic rhinitis, and they often present with markedly elevated eosinophil counts and mononeuritis multiplex (simultaneous deficits of two or several peripheral nerves in different areas of the body). Other symptoms include uveitis, conductive hearing loss, and muscle/ joint pain. An eosinophilic gastroenteritis may precede the onset of the other symptoms.
    Answer A is incorrect. Autoantibodies to acetylcholine receptors are not particularly associated with Churg–Strauss syndrome. Myasthenia gravis is characterized by an autoimmune attack on the acetylcholine receptors of the neuromuscular junction between motor neurons and skeletal muscle fi bers.
    Answer C is incorrect. Autoantibodies to oligodendrocytes are not particularly associated with Churg–Strauss syndrome. There is evidence suggesting that multiple sclerosis may be partially caused by autoimmune antibody attack on central nervous system myelin-secreting oligodendrocytes.
    Answer D is incorrect. Autoantibodies to RBCs, which may be found in certain cases of immune hemolytic anemia, are not particularly associated with Churg–Strauss syndrome.
    Answer E is incorrect. Autoantibodies to TSH receptors are not particularly associated with Churg–Strauss syndrome. Graves’ disease is a disorder resulting from IgG-type autoantibodies to the TSH receptor.

    Rate this question:

  • 21. 

    A 32-year-old HIV-positive man with a recent CD4+ cell count of 84/mm³ and a 3-week history of worsening headaches is brought to the emergency department by ambulance because of acute mental status changes. Upon arrival he is noted to have papilledema, a third cranial nerve palsy, and a rigid neck that cannot be fl exed or extended. He subsequently dies from an overwhelming infection involving his nervous system. An autopsy specimen of the patient’s brain is shown in the image. What is the underlying cause of this patient’s symptoms?

    • A.

      Bacterial meningitis

    • B.

      Fungal meningitis

    • C.

      Herpes encephalitis

    • D.

      Mycobacterial meningitis

    • E.

      Viral meningitis

    Correct Answer
    D. Mycobacterial meningitis
    Explanation
    21. The correct answer is D. This patient suffered from tuberculous meningitis, which is demonstrated by the characteristic acid-fast bacilli present in the patient’s brain tissue. Immunocompromised patients are at risk for developing tuberculous meningitis, which occurs after the central nervous system (CNS) is seeded with mycobacteria that subsequently produce a thick, gelatinous exudate. This exudate typically collects in the basilar region of the CNS and can cause cranial nerve dysfunction (com- monly cranial nerves III, VI, and VII) as well as obstruction of the basilar cisterns, resulting in obstructive hydrocephalus. The presence of cranial nerve involvement and/or obstructive hydrocephalus portends a poor prognosis.
    Answer A is incorrect. Although bacterial meningitis may occur in immunocompromised patients, this tissue section is not consistent with a bacterial infection. Streptococcus pneumoniae and Neisseria meningitidis, the two most common causes of bacterial meningitis, are cocci-shaped organisms.
    Answer B is incorrect. Although fungal meningitis may occur in immunocompromised patients, this tissue section is not consistent with a fungal infection.
    Answer C is incorrect. Although herpes encephalitis may occur in immunocompromised patients, this tissue section is not consistent with a viral herpetic infection.
    Answer E is incorrect. Although viral meningitis may occur in immunocompromised patients, this tissue section is not consistent with a viral infection.

    Rate this question:

  • 22. 

    A 32-year-old female dialysis patient visits her general internist for a health maintenance visit. She subsequently has a dual-energy x-ray absorption examination, which demonstrates signifi cant osteoporosis. What is the most likely etiology of this patient’s osteoporosis?

    • A.

      Chronic metabolic alkalosis

    • B.

      1,25-Dihydroxycholecalciferol excess

    • C.

      Hypercalcemia

    • D.

      Hypophosphatemia

    • E.

      Secondary hyperparathyroidism

    Correct Answer
    E. Secondary hyperparathyroidism
    Explanation
    22. The correct answer is E. Patients with significant renal disease are at particularly high risk for developing skeletal complications, generally known as renal osteodystrophy. Renal failure produces numerous downstream consequences that affect bone health, including increased phosphate retention (resulting in calcium phosphate deposition leading to hypocalcemia and secondary hyperparathyroidism), decreased renal conversion of 25-hydroxcholecalciferol to 1,25-dihydroxycholecalciferol (resulting in decreased intestinal calcium absorption and decreased suppression of parathyroid hormone production), and chronic metabolic acidosis (resulting in increased bone reabsorption). The resulting secondary hyperparathyroidism increases osteoclast activity and the reabsorption of bone.
    Answer A is incorrect. Patients with renal failure have a chronic metabolic acidosis due to decreased renal handling of acid anions. Metabolic alkalosis does not result in osteoporosis.
    Answer B is incorrect. Patients with renal failure have decreased levels of 1,25-dihydroxycholecalciferol because of decreased renal conversion of 25-hydroxycholecalciferol to 1,25-dihydroxycholecalciferol. 1,25-Dihydroxycholecalciferol excess does not result in osteoporosis.
    Answer C is incorrect. Patients with renal failure have hypocalcemia as a result of decreased intestinal absorption of calcium and increased calcium phosphate deposition in tissues. Hypercalcemia is not associated with renal failure and does not result in osteoporosis.
    Answer D is incorrect. Patients with renal failure have hyperphosphatemia due to decreased renal excretion of phosphorous. Hypophosphatemia does not result in osteoporosis.

    Rate this question:

  • 23. 

    A 62-year old man arrives at his doctor’s offi ce complaining of recent onset dull pain in his left fl ank region. He is a retired steel plant worker with a long history of excessive smoking, hypertension, and obesity. He does not recall any history of similar illness in his family. On physical examination a fi rm, homogeneous, nontender movable mass is palpated deep in the left umbilical region near the lower pole of the kidney. Laboratory tests show hypercalcemia, hypophosphatemia, and moderate polycythemia. Urinalysis reveals the presence of hematuria. Which of the following is the most likely diagnosis?

    • A.

      Adult polycystic kidney disease

    • B.

      Angiomyolipoma

    • C.

      Pheochromocytoma

    • D.

      Renal cell carcinoma

    • E.

      Wilms’ tumor

    Correct Answer
    D. Renal cell carcinoma
    Explanation
    .
    23. The correct answer is D. This patient most likely suffers from renal cell carcinoma (RCC). RCC is characterized by the triad of flank pain, hematuria, and abdominal mass, although

    Rate this question:

  • 24. 

    A 55-year-old woman presents to her primary care physician with complaints of nausea, fatigue, early satiety, and abdominal distension. Physical examination reveals the presence of ascites as well as signifi cant pelvic discomfort. CT scanning reveals the presence of multiple masses spread diffusely throughout the abdomen. The patient is subsequently taken to surgery to reduce her tumor burden and to confi rm a diagnosis of metastatic ovarian cancer. Her oncologist then places her on a treatment regimen that includes paclitaxel. Which of the following characterizes the mechanism of this chemotherapy agent?

    • A.

      Alkylating agent that covalently links DNA

    • B.

      Binds tubulin and hyperstabilizes polymerized microtubules

    • C.

      Depolymerizes microtubules

    • D.

      Inhibits DNA polymerase

    • E.

      Inhibits topoisomerase II

    Correct Answer
    B. Binds tubulin and hyperstabilizes polymerized microtubules
    Explanation
    24. The correct answer is B. Paclitaxel (Taxol) binds tubulin and hyperstabilizes polymerized microtubules, thus preventing anaphase. It is derived from the pacific yew tree (Taxus brevifolia), and is currently used with carboplatin as first-line therapy for metastatic ovarian carcinoma. It is also used to treat metastatic adenocarcinoma of the breast.
    Answer A is incorrect. Cyclophosphamide and ifosfamide are alkylating agents that covalently link DNA and are useful in the treatment of non-Hodgkin’s lymphoma and breast and ovarian carcinomas. Cisplatin and carboplatin are platinum-derived compounds that are also thought to act like alkylating agents; they are used with paclitaxel for primary chemotherapy for ovarian cancer, and are also effective for treatment of testicular, bladder, and lung carcinomas.
    Answer C is incorrect. Colchicine depolymerizes microtubules and is derived from the autumn crocus (Colchicum autumnale). It is used in the treatment of acute gout attacks.
    Answer D is incorrect. Cytarabine inhibits DNA polymerase and is useful in the treatment of acute myelogenous leukemia.
    Answer E is incorrect. Etoposide inhibits topoisomerase II and is derived from a springtime herb (Podophyllum peltatum). It is used in the treatment of oat cell and testicular carcinomas.

    Rate this question:

  • 25. 

    A 7-year-old girl has numerous vesicles on her face, particularly around her mouth. Over a few days, the vesicles turn into pustules and crust over, becoming fl aky and light yellow in color. Which of the following statements about the organism most likely responsible for this girl’s infection is correct?

    • A.

      Sabouraud’s agar is required to culture this bacterium

    • B.

      The bacterium is a facultative intracellular organism

    • C.

      The bacterium is a group B β-hemolytic organism

    • D.

      The bacterium is bacitracin-sensitive

    • E.

      The bacterium is protected from host defenses by protein M

    Correct Answer
    D. The bacterium is bacitracin-sensitive
    Explanation
    25. The correct answer is D. This girl has impetigo, caused by Streptococcus pyogenes, a gram positive group A β-hemolytic organism that is bacitracin-sensitive. This infection is characterized by an eruption of vesicles on the face. These vesicles later turn into pustules with a characteristic honey-colored crust. A distinctly bullous form of impetigo is caused by S.aureus.
    Answer A is incorrect. Sabouraud’s agar is required to culture fungi, not Streptococcus pyogenes.
    Answer B is incorrect. Mycobacterium, Brucella, Francisella, Listeria, Yersinia, Legionella, and Salmonella are facultative intracellular organisms, but Streptococcus pyogenes is not. Answer C is incorrect. The bacterium is a group A β-hemolytic organism. Streptococcus agalactiae is a group B β-hemolytic organism.
    Answer E is incorrect. Protein M protects the bacterium from phagocytosis, but it makes the bacterium more sensitive to host defenses due to antibody production against protein M.

    Rate this question:

  • 26. 

    An 18-year-old man has a history of a pancreatic cyst and multiple hemangioblastomas. Which of the following tumors is he at an increased risk of developing?

    • A.

      Acute lymphocytic leukemia

    • B.

      Colon cancer

    • C.

      Pheochromocytoma

    • D.

      Renal cell carcinoma

    • E.

      Testicular seminoma

    Correct Answer
    D. Renal cell carcinoma
    Explanation
    26. The correct answer is D. Patients with VHL disease have hemangioblastomas, or cavernous hemangiomas of the retina, cerebellum, and medulla. They can also present with adenomas and cysts of the liver, kidneys, and pancreas. Patients with VHL disease are also at increased risk of developing renal cell carcinoma.
    Answer A is incorrect. Patients with Down’s syndrome, or trisomy 21, are at increased risk of developing acute lymphocytic leukemia.
    Answer B is incorrect. Patients with familial adenomatous polyposis develop colon cancer if the polyps are not resected.
    Answer C is incorrect. Patients with neurofibromatosis type 1 are at increased risk of developing pheochromocytoma and malignancies such as Wilms’ tumor, rhabdomyosarcoma, and leukemia.
    Answer E is incorrect. Patients with androgen insensitivity are normal-appearing females but can have undescended (inguinal) testicles. If these are not excised, the risk of malignancy, including testicular seminoma, is greatly increased.

    Rate this question:

  • 27. 

    A 42-year-old woman comes to the physician with severe itching for the past 4 days. Her physical examination is signifi cant for hepatomegaly and three xanthomas on her right lower extremity. Laboratory studies reveal a normal total bilirubin level and elevated serum cholesterol and alkaline phosphatase levels. A liver biopsy shows granulomatous destruction of medium- sized intrahepatic bile ducts. Which of the following autoantibodies is most likely to be signifi cantly elevated in this patient?

    • A.

      Anticentromere antibodies

    • B.

      Antihistone antibodies

    • C.

      Antimitochondrial antibodies

    • D.

      Antinuclear antibodies

    • E.

      Rheumatoid factor

    Correct Answer
    C. Antimitochondrial antibodies
    Explanation
    27. The correct answer is C. This patient most likely has primary biliary cirrhosis, which is a chronic autoimmune liver disorder that may initially present with severe itching, hepatomegaly, and xanthomas (yellow nodules/plaques filled with lipid-laden histiocytes, often a sign of hypercholesterolemia). Jaundice develops later in the disease, as hyperbilirubinemia does not occur until there is marked liver damage. The pathologic hallmark is described by the liver biopsy results in the question stem. Over 90% of patients have elevated levels of antimitochondrial antibodies.
    Answer A is incorrect. While patients with primary biliary cirrhosis may also have other autoimmune disorders, the association with anticentromere antibodies is not as strong as the association with antimitochondrial antibodies. It is worth noting that the presence of elevated levels of anticentromere antibodies is often associated with the CREST variant of scleroderma.
    Answer B is incorrect. While patients with primary biliary cirrhosis may also have other autoimmune disorders, the association with antihistone antibodies is not as strong as the association with antimitochondrial antibodies. It is worth noting that the presence of antihistone antibodies is often associated with drug induced lupus syndromes.
    Answer D is incorrect. While patients with primary biliary cirrhosis may also have other autoimmune disorders, the association with antinuclear antibodies is not as strong as the association with antimitochondrial antibodies.
    Answer E is incorrect. While patients with primary biliary cirrhosis may also have other autoimmune disorders, the association with rheumatoid factor is not as strong as the association with antimitochondrial antibodies. It is worth noting that the presence of elevated levels of rheumatoid factor is often associated with rheumatoid arthritis.

    Rate this question:

  • 28. 

    A 25-year-old woman comes to her family physician for a routine check-up. Her physical examination shows a mildly overweight woman but is otherwise unremarkable. A fasting lipid panel, however, shows an LDL cholesterol level of 310 mg/dL, HDL cholesterol level of 42 mg/ dL, triglyceride level of 150 mg/dL, and total cholesterol level of 382 mg/dL. Because a diagnosis of familial hypercholesterolemia is suspected, the doctor initiates treatment of her condition. Soon after starting treatment, however, she presents with myalgias. Laboratory values show elevated levels of aspartate aminotransferase, alanine aminotransferase, and creatinine kinase. Which of the following interventions is most likely responsible for the patient’s myalgias?

    • A.

      B complex vitamins

    • B.

      Hormone replacement therapy

    • C.

      LDL apheresis

    • D.

      Liver transplant

    • E.

      Statin medications

    Correct Answer
    E. Statin medications
    Explanation
    28. The correct answer is E. Dietary modification (drastically limiting saturated and trans fats and cholesterol), weight loss, and aerobic exercise are the first-line treatment options for any patient with elevated cholesterol levels, and these lifestyle modifications should be attempted by this patient but will likely have only a minimal effect in a patient with familial hypercholesterolemia. Statin medications are 3-hydroxy-3-methylglutaryl coenzyme A reductase inhibitors. By blocking the rate-limiting step in cholesterol synthesis, they can lower cholesterol levels. High-dose therapy with a statin such as atorvastatin, or combined therapy with one of the fibrate drugs, is usually initiated. In terms of toxicity, statins can potentially cause myositis, which is causing this patient’s symptoms as well as her elevated creatinine kinase levels. Rarely, patients can develop rhabdomyolysis with renal failure. Elevated liver enzyme levels can also be observed with statin treatment, although this is usually reversible.
    Answer A is incorrect. Niacin (vitamin B3) can lower LDL and increase HDL levels. However, statin medications are first-line therapy for this disorder. Niacin can be added to the regimen as a third drug (with a fibrate) or can be used in patients who are refractory to statin treatment. Its use is often limited by tolerability (it causes flushing in the majority of patients).
    Answer B is incorrect. Hormone replacement therapy is contraindicated in patients with high cholesterol due to the increased risk of heart attack and stroke. Also, hormone replacement therapy is not necessary for a woman in her 20s.
    Answer C is incorrect. LDL apheresis is a method for selectively removing LDL molecules from the blood using immunoadsorption columns. This process takes at least 3 hours and is done every 1–2 weeks. It is very expensive and not readily available.
    Answer D is incorrect. Liver transplantation can dramatically lower LDL levels in patients with familial hypercholesterolemia by providing them with normal LDL receptors. However, the risks associated with transplantation make this option a last resort.

    Rate this question:

  • 29. 

    Informed consent is the legal demonstration of a patient’s understanding of risks, benefi ts, and outcomes of treatments and alternatives. Which of the following circumstances represents an exception for obtaining informed consent?

    • A.

      A competent patient whose son wishes to waive his father’s right

    • B.

      A mentally retarded patient who is legally competent

    • C.

      A nursing volunteer has already obtained informed consent

    • D.

      A paralyzed patient who cannot speak but can nod his head for consent

    • E.

      A physician agrees that informing the patient will be detrimental to the patient’s health

    Correct Answer
    E. A physician agrees that informing the patient will be detrimental to the patient’s health
    Explanation
    29. The correct answer is E. The therapeutic privilege is a rare case of an appropriate exception to informed consent. The principle is that informing the patient will not be medically sound for the patient. In general, a physician should consult another physician not involved in the patient’s care, a psychiatrist, and/or an ethics committee when invoking this principle. It does not refer to withholding information a physician believes will make a patient less likely to have a procedure performed.
    Answer A is incorrect. Only a patient can waive their own right to informed consent.
    Answer B is incorrect. Any competent adult can make informed consent.
    Answer C is incorrect. Usually, physicians must obtain informed consent. In some instances, a qualified nurse or other medical professional may obtain informed consent; however a nursing volunteer can certainly not.
    Answer D is incorrect. A competent patient may acknowledge informed consent by any means.

    Rate this question:

  • 30. 

    A 43-year-old man presents to the emergency department with the sudden onset of a headache, sweating, and palpitations. On physical examination, he appears anxious and his face is fl ushed. His blood pressure is 150/95 mm Hg. Urinalysis reveals high levels of vanillylmandelic acid. Which of the following is the most appropriate pharmacologic treatment for this patient’s condition?

    • A.

      Phenelzine

    • B.

      Phenoxybenzamine

    • C.

      Phenylephrine

    • D.

      Propranolol

    Correct Answer
    B. Phenoxybenzamine
    Explanation
    30. The correct answer is B. This patient has the classic presentation of pheochromocytoma, a tumor of the adrenal medulla that causes hypersecretion of catecholamines. Patients tend to experience sudden “spells” of elevated blood pressure, palpitations, headache, diaphoresis, and anxiety. Diagnosis of this disorder is made by demonstration of elevated urinary excretion of catecholamines or their metabolites, metanephrines and vanillylmandelic acid. Pharmacologically, this disorder is managed with nonselective α-antagonists (phenoxybenzamine and phentolamine). It is also useful to keep in mind that pheochromocytoma is a part of multiple endocrine neoplasma (MEN) type II (medullary carcinoma of the thyroid, pheochromocytoma, and parathyroid tumor) and MEN type III (medullary carcinoma of the thyroid, pheochromocytoma, and mucosal neuromas).
    Answer A is incorrect. Phenelzine, a monoamine oxidase inhibitor, is used to treat depression is in fact contraindicated in patients with pheochromocytoma because it can exacerbate hypertension.
    Answer C is incorrect. Phenylephrine, an α-agonist, would not be appropriate for the treatment of pheochromocytoma because it would worsen the patient’s hypertension via its vasoconstrictive effect at α1-receptors.
    Answer D is incorrect. Propranolol, a nonselective β-blocker, is not indicated for the treatment of pheochromocytoma because its actions are limited to β receptors. Thus, it would have little effect in antagonizing the α1-agonist actions of norepinephrine that are of concern in these tumors.

    Rate this question:

  • 31. 

    A 78-year-old man comes to the physician for evaluation after falling fi ve times in 2 months. An x-ray skeletal survey reveals no fractures, but the patient admits to worsening urinary incontinence over the previous 4 months. His wife states that his memory and concentration have deteriorated recently. The patient’s vital signs are normal, and his physical examination is notable for a wide-based gait with short steps. A Mini-Mental State Examination results in a score of 26/30. His funduscopic examination is normal, and his neurologic examination is notable for slight bradykinesia without tremor. Laboratory tests, including serum vitamin B12, folate, and TSH, are normal. What is the most likely etiology of this patient’s recent decline?

    • A.

      Alzheimer’s disease

    • B.

      Hypothyroidism

    • C.

      Multi-infarct dementia

    • D.

      Normal pressure hydrocephalus

    • E.

      Parkinson’s disease

    Correct Answer
    D. Normal pressure hydrocephalus
    Explanation
    31. The correct answer is D. This patient has a potentially reversible case of dementia: normal pressure hydrocephalus (NPH), with the classic triad of incontinence, gait difficulty, and mental decline (“wet, wobbly, and wacky”). Patients with NPH often demonstrate mild bradykinesia and their gait has been described as “magnetic” because their feet seemingly cling to the floor. The score of 26/30 on the Mini- Mental State Examination indicates only that some mild abnormality is present. Regardless, the patient should undergo magnetic resonance imaging to rule out a mass lesion that could cause similar symptoms. The pathophysiology of NPH is not well understood, but it is thought that neurons are stretched secondary to ventricular dilation caused by excessive cerebrospinal fluid production, decreased absorption, or both. It is imperative to identify these patients because timely intervention with a ventriculoperitoneal shunt can reverse the dementia and decline.
    Answer A is incorrect. Alzheimer’s disease can present with some of the symptoms in this case. However, significant physical impairment tends to occur later in the Alzheimer’s disease process and would thus correlate with a much lower score on the Mini-Mental State Examination. The time course and the relatively rapid progression in symptoms are not consistent with this diagnosis.
    Answer B is incorrect. Hypothyroidism, another potential cause of reversible dementia in the elderly, should be ruled out early in the work-up. This patient’s TSH level is normal, indicating euthyroidism.
    Answer C is incorrect. Multi-infarct dementia is the most common cause of cognitive decline with a stepwise drop in function in the setting of prior cerebrovascular disease and stroke. In this case, the decline has been steadily progressive in a patient with no history of vascular disease.
    Answer E is incorrect. Parkinson’s disease classically presents with bradykinesia, masklike facies, shuffling gait, tremor, and rigidity. This patient has mild bradykinesia and no rigidity or tremor, so this diagnosis is a less likely possibility.

    Rate this question:

  • 32. 

    An 8-month-old girl is brought to the emergency department by her parents because she appears swollen. The parents weaned the child from formula 3 weeks ago. They have been giving her rice milk for 1 month. On examination the child has a protuberant belly and 2+ pitting edema in her wrists and shins. This type of malnutrition is caused by a defi ciency of what type of nutrient?

    • A.

      Calorie

    • B.

      Carbohydrate

    • C.

      Electrolyte

    • D.

      Fat

    • E.

      Protein

    Correct Answer
    E. Protein
    Explanation
    32. The correct answer is E. Protein malnutrition, or kwashiorkor, is characterized by an inadequate intake of protein but adequate intake of calories. Edema is the most common presenting symptom, but depigmented hair, anorexia, fatty liver changes, and skin lesions are also seen. Edema is caused by low levels of protein that decrease the plasma oncotic pressure and result in a loss of fluid into interstitial spaces. This patient’s history and presentation are consistent with a low-protein, normal-calorie diet.
    Answer A is incorrect. Marasmus describes caloric deficiency that can occur even with an adequate protein intake. Like kwashiorkor, marasmus is seen in infants at times of diet change such as weaning from breast milk in areas of the world where food can be scarce. However, marasmus is a generalized starvation with loss of body fat and protein. The symptoms of marasmus are muscle wasting, weakness, arrested growth, and anemia.
    Answer B is incorrect. When the intake of carbohydrates is low, amino acids are deaminated to provide carbon skeletons for gluconeogeneis. However, this will not produce edema as seen in kwashiorkor.
    Answer C is incorrect. Electrolyte deficiencies have various effects, but some are clinically difficult to recognize in an 8-month-old infant. For example, hyponatremia can manifest with confusion, muscle cramps, anorexia, nausea, seizures, and coma; hypokalemia can manifest with nausea, vomiting, muscle weakness or paralysis, rhabdomyolysis, and cardiac arrhythmias. However, overt edema from solely an electrolyte deficiency is a highly unlikely presentation.
    Answer D is incorrect. Fat malnutrition would be unlikely in this child based on the presentation. Rice milk has about one-fourth the amount of fat in cow’s milk or soy milk. Fats, especially the essential fatty acids linoleic acid and linolenic acid, are largely obtained in the diet. They are required for the development of cell membranes and nerve cell sheaths. Deficiency in fat causes developmental delays but not edema. In addition, with adequate calories the body can generate most fatty acids.

    Rate this question:

  • 33. 

    A 19-year-old man who recently immigrated from Asia comes to the emergency department because of blood in his sputum. On history, the patient mentions he has had weight loss and night sweats. On examination, the patient has a fever and bronchial breath sounds with crepitant rales. Laboratory tests show lymphocytosis and an increased erythrocyte sedimentation rate. X-ray fi lm of the chest shows a calcifi ed lung lesion and hilar lymphadenopathy. Which of the following is the stain used to identify the most likely infectious organism?

    • A.

      Congo red

    • B.

      Giemsa’s

    • C.

      India ink

    • D.

      Periodic acid-Schiff

    • E.

      Ziehl-Neelsen

    Correct Answer
    E. Ziehl-Neelsen
    Explanation
    33. The correct answer is E. This patient most likely has Mycobacterium tuberculosis infection. M. tuberculosis is an acid-fast bacillus, and Ziehl–Neelsen stain is used to reveal acidfast bacteria. Characteristics favoring a diagnosis of tuberculosis include immigrant status, night sweats, weight loss, and chest x-ray findings. Primary tuberculosis is known to result in Ghon complexes, which show up as calcifications on x-ray imaging. Ghon complexes are a combination of parenchymal lesions and involved hilar and/or mediastinal lymph nodes. The lesions are calcified because of the caseating granuloma formation. Secondary tuberculosis presents with cavitary lesions and is seen more in immunocompromised patients. Other pathologies that can present with hilar/mediastinal nodes are lymphoma and sarcoidosis, making the Gram stain important in diagnosis. M. tuberculosis is aerobic and gram positive.
    Answer A is incorrect. Congo red is used to visualize amyloid, showing apple-green birefringence in polarized light.
    Answer B is incorrect. Giemsa’s stain is useful for Borrelia, Plasmodium, trypanosomes, and Chlamydia.
    Answer C is incorrect. India ink is the stain of choice for Cryptococcus neoformans.
    Answer D is incorrect. Periodic acid-Schiff stains glycogen, and thus it can be used to visualize mucopolysaccharides. In addition, is used to diagnose Whipple’s disease.

    Rate this question:

  • 34. 

    A 65-year-old postmenopausal woman presents with progressive constipation and frequent, excessive urination. She mentions she has been smoking 1 pack per day since she was 19 years old. On further questioning she states she is having palpitations constantly. On physical examination, there are respiratory fi ndings, which prompt an x-ray fi lm of the chest. A circular, perihilar lesion in the lung is found. Laboratory testing shows a decreased phosphorus level. Which of the following is the most likely cause of this patient’s symptoms?

    • A.

      Central bronchogenic carcinoma

    • B.

      Cervical sympathetic chain compression

    • C.

      Chronic silica exposure

    • D.

      Congenital chloride channel dysfunction

    • E.

      Dynein arm defect in cilia

    • F.

      Ectopic antidiuretic hormone production

    • G.

      Solitary parathyroid adenoma

    Correct Answer
    A. Central bronchogenic carcinoma
    Explanation
    34. The correct answer is A. The patient has symptoms of hypercalcemia. With a history of smoking and “coin” lesion in the lung, one should suspect a lung tumor that produces parathyroid hormone-related peptide. Squamous cell carcinoma is a centrally located bronchogenic carcinoma.
    Answer B is incorrect. Horner’s syndrome is characterized by ptosis, miosis, and anhidrosis. It is a complication of lung cancer at the apex, referred to as a Pancoast tumor.
    Answer C is incorrect. Chronic silica exposure is associated with increased tuberculosis susceptibility. Tuberculosis usually presents with chronic cough, hemoptysis, fevers, chills, and weight loss.
    Answer D is incorrect. Cystic fibrosis causes respiratory, reproductive, and gastrointestinal symptoms.
    Answer E is incorrect. Kartagener’s syndrome is associated with sinusitis, bronchiectasis, and infertility.
    Answer F is incorrect. ADH, which one could see with small-cell carcinoma, would cause water retention (oliguria) and fatigue.
    Answer G is incorrect. Solitary parathyroid adenoma can present with hypercalcemia and low phosphorus levels. However, the patient’s smoking history and new findings on chest imaging cannot be ignored.

    Rate this question:

  • 35. 

    The image shows a specialized epithelium that overlies a type of peripheral lymphoid tissue. What is the main class of antibodies associated with this lymphoid tissue?

    • A.

      IgA

    • B.

      IgD

    • C.

      IgE

    • D.

      IgG

    • E.

      IgM

    Correct Answer
    A. IgA
    Explanation
    35. The correct answer is A. The image depicts the epithelium that lies above the Peyer’s patches, lymphoid tissues found within the ileum. This gut-associated lymphoid tissue is a vital collection of peripheral lymphoid tissue. Note that on a microscopic image of Peyer’s patches, this epithelium has several M cells, specialized cells that function to endocytose and phagocytose particles in the lumen of the gut. Thus, they serve as immune surveillance in the intestines. In adults, B lymphocytes predominate in Peyer’s patches and secrete IgA, the main antibody present within the mucosal lining of the gut. It is synthesized by plasma cells that reside within the lamina propria. Of note is the fact that several gut pathogens express virulence factors, known as IgA proteases, which cleave and therefore deactivates the dimeric IgA antibodies.
    Answer B is incorrect. IgD is found only on the surface of B lymphocytes; its function is not known.
    Answer C is incorrect. IgE orchestrates the type I hypersensitivity response.
    Answer D is incorrect. IgG is the main antibody produced during a secondary immune response and also the most abundant.
    Answer E is incorrect. While IgM can be found within the gut lumen, IgA predominates.

    Rate this question:

  • 36. 

    A 45-year-old man with essential hypertension presents to the emergency department with muscle weakness and palpitations. Peaked T waves and prolonged PR intervals are noted on ECG. Which of the following medications could be the underlying cause of the changes noted on this patient’s ECG?

    • A.

      Acetazolamide

    • B.

      Furosemide

    • C.

      Hydrochlorothiazide

    • D.

      Mannitol

    • E.

      Spironolactone

    Correct Answer
    E. Spironolactone
    Explanation
    36. The correct answer is E. Peaked T waves and prolonged PR intervals are evident on ECG in cases of hyperkalemia. Spironolactone is a potassium- sparing diuretic that inhibits K+ secretion at the cortical collecting duct; therefore, an important side effect of spironolactone use is hyperkalemia.
    Answer A is incorrect. Acetazolamide inhibits carbonic anhydrase at the proximal convoluted tubule to cause increased excretion of HCO3 –. It has no effect on potassium concentrations and thus would not result in hyperkalemia.
    Answer B is incorrect. As a loop diuretic, furosemide inhibits the Na+-K+-2Cl– cotransporter in the thick ascending loop of Henle and causes secretion of potassium from the kidney. Hypokalemia can result.
    Answer C is incorrect. Thiazide diuretics inhibit an Na+-Cl– cotransporter in the distal tubule. Increased distal nephron filtrate flow and activation of the renin-angiotensin-aldosterone axis cooperate to encourage potassium loss during thiazide diuresis. Thus, hypokalemia is an adverse effect of excess hydrochlorothiazide.
    Answer D is incorrect. Mannitol is an osmotic diuretic that increases urine output by drawing fluid into the filtrate. It has no effect on ion transport proteins and thus does not change potassium excretion.

    Rate this question:

  • 37. 

    A physician is caring for a hospitalized 31-yearold man with long-standing, poorly controlled type 1 diabetes mellitus. He is blind and has peripheral neuropathy with sensory loss in both feet, and his most recent hemoglobin A1c level was 13.9%. He recently presented with altered mental status, polyuria, and polydipsia. At that time, his serum glucose level was 475 mg/dL, arterial blood pH was 6.96, and his anion gap was 27. Since then his acidosis has resolved with appropriate treatment, and fi ngerstick blood glucose levels have normalized. However, he has persistent nasal discharge; paranasal sinus tenderness; and new onset of periorbital edema, proptosis, facial numbness, and obtundation. Fungal stain of fl uid obtained from urgent surgical sinus drainage would most likely reveal which of the following?

    • A.

      Broad-based budding dimorphic fungi

    • B.

      45-degree angle branching, septate hyphae with rare fruiting bodies

    • C.

      Irregular, broad, nonseptate hyphae with 90-degree branching

    • D.

      Pseudohyphae with budding yeasts

    • E.

      5- to 10-μm yeasts with wide capsular halo on India ink stain

    Correct Answer
    C. Irregular, broad, nonseptate hyphae with 90-degree branching
    Explanation
    37. The correct answer is C. A feared infectious complication seen in patients with long-standing diabetic ketoacidosis is invasive rhinocerebral mucormycosis. As in this case, this infection leads to persistent sinusitis with inevitable invasion into adjacent neural structures such as the trigeminal nerve and the frontal lobe. Rhizopus organisms thrive in serum containing high glucose levels and low pH. Other conditions predisposing patients to this aggressive infection include iron overload/chelator treatment, AIDS, immunosuppression due to prolonged steroid use, and hematologic malignancies. Under the microscope, Mucor species appear as irregular, broad, nonseptate hyphae with 90-degree branching. Both Mucor and Rhizopus species can cause this condition.
    Answer A is incorrect. Blastomycetes species appear as broad-based, budding, dimorphic fungi. Blastomycosis is mainly a pulmonary infection, endemic to states east of the Mississippi River and Central America. Infection of the lung leads to polygranulomatous infection with frequent hematogenous dissemination. These fungal species are cultured on Sabouraud’s agar.
    Answer B is incorrect. Aspergillus species appear microscopically as 45-degree angle branching, septate hyphae with rare fruiting bodies. Invasive Aspergillus infection occurs mainly after prolonged profound immunosuppression (as in patients with AIDS or cancer or in individuals with chronic granulomatous disease) and typically leads to bronchopulmonary aspergillosis with cavitary lesions in the lung.
    Answer D is incorrect. Candidal species appear microscopically as pseudohyphae with budding yeasts. Candidal infections are more common in patients with poorly controlled diabetes, but they rarely cause invasive rhinocerebral infections as described in this patient. Rather, candidal infection may cause vulvovaginosis, chronic mucocutaneous infections, or disseminated candidiasis in advanced cases.
    Answer E is incorrect. Cryptococcus species appear as 5- to 10-μm yeasts with a wide capsular halo. These infections represent an important opportunistic infection in patients with AIDS, causing cryptococcal meningitis. Cryptococcal infections are extremely rare in patients with normal CD4+ T-lymphocyte counts.

    Rate this question:

  • 38. 

    A 64-year-old woman presents to her primary care physician with fatigue, weakness, and a weight loss of 4.5 kg (10 lb) in the past 4 months. Additionally, she notes that her vision has deteriorated over that time, and has had several severe nosebleeds. Physical examination demonstrates hepatosplenomegaly, and laboratory tests show an increased total protein level. Serum protein electrophoresis reveals a large spike in the gamma region. A skeletal survey is negative. Which of the following is the most likely diagnosis?

    • A.

      Chronic lymphocytic leukemia

    • B.

      Diabetes mellitus

    • C.

      Monoclonal gammopathy of undetermined signifi cance

    • D.

      Multiple myeloma

    • E.

      Waldenström’s macroglobulinemia

    Correct Answer
    E. Waldenström’s macroglobulinemia
    Explanation
    38. The correct answer is E. The disease that is described is Waldenstrom’s macroglobulinemia, which is characterized by weakness, weight loss, a monoclonal M spike on serum protein electrophoresis (seen as a large spike in the gamma region), and a hyperviscosity syndrome (manifesting as nosebleeds, headaches, and vision disturbances). Hyperviscosity is caused by the large amount of IgM protein in the blood produced by a B-cell neoplasm. These large proteins interfere with microvascular and cellular processes, causing blood vessel damage, which results in headaches due to impaired cranial blood flow and in disturbances in vision due to poor ocular blood flow. Additionally, circulating IgM proteins can bind to clotting factors and inhibit them, causing increased bleeding.
    Answer A is incorrect. Chronic lymphocytic leukemia (CLL) typically presents with lymphadenopathy, hepatosplenomegaly, a warm antibody autoimmune hemolytic anemia, and smudge cells in the peripheral blood. The hyperviscosity syndrome is not present in CLL.
    Answer B is incorrect. Diabetes presents with nocturia, polyuria, and polydipsia. Blood tests would demonstrate increased glucose. Superficial resemblances between the hyperviscosity syndromes and diabetic retinopathy, and diabetic kidney disease with the renal insufficiency of multiple myeloma, may be misleading. However, bleeding complications due to diabetes alone would be rare.
    Answer C is incorrect. Monoclonal gammopathy of undetermined significance (MGUS) is similar to the condition described above in that it, too, has a monoclonal spike. An important difference is that MGUS is asymptomatic due to a lower level of protein. Some patients may experience mild polyneuropathy, but they will not have the bone pain, renal failure, and anemia of multiple myeloma or the hyperviscosity of Waldenstrom’s macroglobulinemia. Nonetheless, MGUS may be a premalignant lesion that can progress to multiple myeloma.
    Answer D is incorrect. Multiple myeloma is similar to the condition described above and it also involves abnormal plasma cells overproducing immunoglobulin, seen as a monoclonal M spike (critical for diagnosis). However, instead of a hyperviscosity syndrome, multiple myeloma typically presents with a collection of other characteristic symptoms. These symptoms include with lytic bone lesions causing bone pain and hypercalcemia, renal insufficiency and azotemia, increased susceptibility to infection, and anemia. Additionally, one may find Bence Jones protein (Ig light chains) in the urine and a rouleaux formation of RBCs on peripheral blood smear.

    Rate this question:

  • 39. 

    A pharmaceutical company has created a new drug that, when taken daily, is thought to be highly effective at preventing the onset of migraines. The company would like to market the drug and is conducting a study to look at its benefi ts and possible risks. In coordination with a physician at a local hospital, it enrolls 800 people for the study. The physician places 100 patients with the worst and most frequent migraines in the medication group, as he thinks that they are in most need of the drug’s benefi t. Which of the following best explains why the drug may not perform up to expectations?

    • A.

      Differences in group size

    • B.

      Late-look bias

    • C.

      Recall bias

    • D.

      Sampling bias

    • E.

      Selection bias

    Correct Answer
    E. Selection bias
    Explanation
    39. The correct answer is E. Selection bias is being displayed in this scenario. The physician is selecting his more serious cases for the treatment group (i.e., those who are in most need of the benefit). The placebo group contains patients who are healthier, less symptomatic, and more likely to have a better outcome. Therefore, when it comes time for collecting data, the drug’s beneficial effect compared to placebo may be blunted.
    Answer A is incorrect. Studies can still be valid if there are differences in group size. There is no evidence that there is a difference in group size in this scenario.
    Answer B is incorrect. Late-look bias occurs when information or results are gathered at an inappropriate time. Late-look bias is not displayed in this scenario.
    Answer C is incorrect. Recall bias occurs when knowledge of the presence of a disorder alters the way a subject remembers his or her history. For example, a patient may over- or underestimate his or her consumption of a certain drug upon learning of its detrimental effect to the body. Recall bias is not displayed in this scenario.
    Answer D is incorrect. Sampling bias occurs when those in the trial are not truly representative of the general population. Therefore, the results (both positive and negative) of the study cannot be truly applied to the general population. There is no evidence of sampling bias in this scenario.

    Rate this question:

  • 40. 

    The human papillomavirus promotes neoplasia through production of the viral proteins E6 and E7, which interfere with the normal function of Rb and p53. What general function is common to both Rb and p53?

    • A.

      Cell cycle regulation

    • B.

      Cellular adhesion

    • C.

      Direct transcriptional control

    • D.

      DNA repair

    • E.

      Inhibition of signal transduction

    Correct Answer
    A. Cell cycle regulation
    Explanation
    40. The correct answer is A. This question addresses mechanisms of neoplastic transformation via loss of tumor suppressors. Each of the choices is a valid tumor suppressor function which, if lost, promotes malignancy. Rb and p53 act as red flags that halt the cell cycle if, for example, DNA is damaged.
    Answer B is incorrect. E-cadherin is a tumor suppressor that is involved in cellular adhesion. Some breast and stomach cancers are associated with mutations in e-cadherin.
    Answer C is incorrect. WT-1, a nuclear transcription factor, is mutated in Wilms’ tumor. p53 is a transcription factor, but Rb interacts only with transcription factors, thereby indirectly affecting gene expression.
    Answer D is incorrect. The breast cancerassociated genes BRCA1 and BRCA2 are involved in DNA repair.
    Answer E is incorrect. Tumor suppressors such as APC (gastrointestinal cancers and melanoma) and NF1 (neurofibromatosis type 1) inhibit signal transduction.

    Rate this question:

  • 41. 

    A 95-year-old woman is transferred to the intensive care unit after a 3-day history of cough and declining mental status. Her blood pressure is 85/50 mm Hg, pulse is 124/min, temperature is 39.8° C (103.6° F), and respiratory rate is 27/min. Crackles are heard at the left lower lung base, and the patient is suffering from rigors intermittently. Blood and sputum cultures drawn at the onset of symptoms grow strains of Klebsiella pneumoniae resistant to all antibiotics except polymyxin B. Which of the following is a serious adverse reaction of polymyxin B?

    • A.

      Granulocytopenia

    • B.

      Hearing loss

    • C.

      Hemolysis in patients with glucose 6- phosphate dehydrogenase defi ciency

    • D.

      Numbness of the extremities

    • E.

      Severe vomiting

    Correct Answer
    D. Numbness of the extremities
    Explanation
    41. The correct answer is D. Polymyxins bind to gram-negative bacterial cell membrane phospholipids and destroy the membrane by acting like a detergent. They have no activity against gram-positive organisms or fungi. Polymyxins are predominantly used to treat severe gramnegative infections that are resistant to less toxic antimicrobials. Polymyxins are rarely used owing to their association with nephrotoxicity and neurotoxicity. Numbness of the extremities is one manifestation of neurotoxicity, but dizziness, drowsiness, confusion, nystagmus, and blurred vision are also possible.
    Answer A is incorrect. Granulocytopenia refers to a low number of granulocytes (neutrophils, eosinophils, and basophils). Certain medications can cause granulocytopenia, most commonly clozapine, ticlopidine, sulfasalazine, and antithyroid drugs. Among antibiotics, trimethoprim and dapsone are most commonly implicated.
    Answer B is incorrect. Ototoxicity is a common adverse effect of aminoglycosides and vancomycin, particularly when the two agents are used in combination. It is not an adverse effect of polymyxins.
    Answer C is incorrect. Patients with glucose 6-phosphate dehydrogenase deficiency are predisposed to hemolysis, and drugs with a high redox potential can precipitate rapid destruction of RBCs. In particular, primaquine and sulfonamides can precipitate a severe anemia characterized by sudden onset of jaundice, pallor, and dark urine with back pain.
    Answer E is incorrect. Vomiting is an adverse effect of many antibiotics, but it is not an adverse effect of polymyxins.

    Rate this question:

  • 42. 

    A 20-year-old woman presents to the physician with a history of bloody diarrhea and abdominal pain. She states that she has not traveled recently or changed her eating habits. A stool culture is negative for any known infectious cause of diarrhea. A fl exible sigmoidoscopy is performed and shows numerous lesions in the descending colon interrupted by normal-appearing mucosa. Which of the following features would most likely be present on a tissue biopsy of the affected region?

    • A.

      Cells with loss of mucin and hyperchromatic nuclei

    • B.

      Hyperplastic goblet cells

    • C.

      Mucus-fi lled cells in crypts

    • D.

      Noncaseating granulomas

    • E.

      Ulcerated mucosa only

    Correct Answer
    D. Noncaseating granulomas
    Explanation
    42. The correct answer is D. Inflammatory bowel disease (IBD) typically presents during late adolescence to early adulthood with symptoms of abdominal pain and frequent bouts of diarrhea. Types of IBD are differentiated and diagnosed on the basis of their clinical picture, their appearance on endoscopy and biopsy, and the exclusion of other intestinal infectious etiologies. In this patient, the areas of normal-appearing mucosa should immediately point to the diagnosis of Crohn’s disease as opposed to ulcerative colitis. Ulcerative colitis is characterized by mucosal inflammation that is limited to the colon and frequently involves the rectum. Crypt abscesses and ulceration of the mucosa are classically seen on biopsy. Crohn’s disease, however, shows transmural inflammation interspersed with normal mucosa (skip lesions), as seen in this patient. It can affect any part of the gastrointestinal tract but usually spares the rectum. Noncaseating granulomas may be found in Crohn’s disease but are not found in ulcerative colitis.
    Answer A is incorrect. Cells with loss of mucin and hyperchromatic nuclei are present in colon cancer, which is more commonly associated with ulcerative colitis.
    Answer B is incorrect. Hyperplasia of goblet cells is the central feature of hyperplastic polyps, the most common type of non-neoplastic polyp. Although usually asymptomatic, they may cause bleeding, abdominal pain, and, rarely, obstruction.
    Answer C is incorrect. Mucus-filled cells in crypts are part of normal colonic mucosa.
    Answer E is incorrect. Ulceration limited to the mucosa is a feature of ulcerative colitis. In Crohn’s disease, the inflammation is often transmural and interspersed with areas of normal-appearing tissue, as described in this vignette.

    Rate this question:

  • 43. 

    A 38-year-old man from rural Guatemala dies in the emergency department due to heart failure. Autopsy reveals an enlarged and fl accid heart. Microscopy of a blood sample taken before the man’s death shows fl agellated parasites. A histologic section of the heart shows amastigotes. Which of the following parasites did this man most likely harbor?

    • A.

      Cryptosporidium species

    • B.

      Entamoeba histolytica

    • C.

      Giardia lamblia

    • D.

      Toxoplasma gondii

    • E.

      Trypanosoma cruzi

    Correct Answer
    E. Trypanosoma cruzi
    Explanation
    43. The correct answer is E. Trypanosoma cruzi infection can cause aganglionic megacolon and Chagas’ disease, a condition in which the heart is enlarged and flaccid. T. cruzi is transmitted via the reduviid bug. Microscopic examination reveals flagellated trypomastigotes in the blood and nonflagellated amastigotes in cardiac muscle. T. cruzi infection is treated with nifurtimox. The fact that this man is from Central America is a second clue to his illness; epidemiologically, T. cruzi infections are most common among the poor in rural Central and South America.
    Answer A is incorrect. Cryptosporidium infection presents with severe diarrhea in HIVpositive patients and mild watery diarrhea in HIV-negative patients. Cryptosporidium is transmitted via cysts in water (fecal-oral transmission). Microscopically, acid-fast staining cysts are found. Unfortunately, there is no treatment available for Cryptosporidium infection; however, in healthy patients, cryptosporidiosis is self-resolving.
    Answer B is incorrect. Entamoeba histolytica infection presents with bloody diarrhea (dysentery), abdominal cramps with tenesmus, and pus in the stool. It can also cause liver abscesses accompanied by right upper quadrant pain. E. histolytica is transmitted via cysts in water (fecal-oral transmission). On microscopy one observes amebas with ingested RBCs. Treatment for E. histolytica infection includes metronidazole and iodoquinol.
    Answer C is incorrect. Giardia lamblia infection presents with bloating, flatulence, foul smelling diarrhea, and light-colored fatty stools. G. lamblia is transmitted via cysts in water (fecal- oral transmission). On microscopy, one observes teardrop-shaped trophozoites with a ventral sucking disc or cysts. Metronidazole is used to treat G. lamblia infection.
    Answer D is incorrect. Toxoplasma gondii infection presents with brain abscesses in HIVpositive patients and with birth defects if infection occurs during pregnancy (toxoplasmosis is one of the ToRCHeS organisms). T. gondii is transmitted via cysts in raw meat or cat feces. The definitive stage (sexual stage) occurs in cats. The diagnosis is most often made serologically. Sulfadiazine and pyrimethamine are used to treat toxoplasmosis.

    Rate this question:

  • 44. 

    A 27-year-old man presents to the emergency department with a cough productive of bloodtinged sputum. He also complains that in the past couple weeks he has noticed increased fatigue and some blood in his urine. A renal biopsy is performed that, upon immunofl uorescence, shows a linear pattern of IgG deposition along the basement membrane. Which of the following is most likely responsible for this patient’s disease?

    • A.

      Antineutrophil cytoplasmic antibodies

    • B.

      Anti-type III collagen antibodies

    • C.

      Anti-type IV collagen antibodies

    • D.

      Immune complexes

    • E.

      T lymphocytes

    Correct Answer
    C. Anti-type IV collagen antibodies
    Explanation
    44. The correct answer is C. A young man presenting with hemoptysis should raise a high index of suspicion for Goodpasture’s syndrome. This diagnosis is supported by his fatigue and hematuria (although typically renal symptoms follow pulmonary symptoms by weeks to months). As the disease progresses, one would expect a nephritic picture with hematuria, hypertension, and oliguria. The diagnosis of Goodpasture’s syndrome is confirmed by the renal biopsy, which on immunofluorescence staining shows a linear pattern of IgG deposition along the basement membrane. These anti–glomerular basement membrane (GBM) antibodies are specific c to the α3 chain of type IV collagen, and cause injury to both the glomerular and alveolar basement membranes.
    Answer A is incorrect. ANCAs are found in certain pauci-immune glomerulonephritides such as Wegener’s granulomatosis. This could account for a nephritic picture, but immunofluorescence would show an absence of any immune deposition. Furthermore, if the patient had Wegener’s granulomatosis, one would expect to see a specific pattern of symptoms involving the sinuses, lungs, and kidneys.
    Answer B is incorrect. Goodpasture’s syndrome is caused by anti-GBM antibodies specific to the α3 chain of type IV collagen. Type III collagen is found in skin, blood vessels, and other organs and is not affected by anti–GBM antibodies. The most common pathology involving type III collagen is Ehlers–Danlos syndrome, a connective tissue disorder in which patients bleed very easily and have hyperelastic skin.
    Answer D is incorrect. Immune complex deposition causes damage to the glomerulus in many diseases such as poststreptococcal glomerulonephritis and systemic lupus erythematosus. The cause can be idiopathic, due to an antigenic stimulus, or due to a systemic immune complex disorder. On immunofluorescence one would see a less even distribution of staining, with lumpy or granular deposition of immune complexes in the glomerulus.
    Answer E is incorrect. The patient’s presentation is characteristic of Goodpasture’s syndrome, which is caused by antibodies specific to type IV collagen. Immune-related injury to the glomerulus can be separated into three categories: immune-complex glomerulonephritis, anti–GBM, and pauci-immune glomerulonephritis (no antibodies, complement, or immune deposition). It has been proposed that pauci-immune glomerulonephritis is mediated by T lymphocytes, which release cytokines and thereby recruit inflammatory cells.

    Rate this question:

  • 45. 

    A 36-year-old man who completed a marathon 6 hours earlier presents to the emergency department with severe muscle pain and swelling and complaints of red urine. Laboratory tests show a creatine kinase level of 6800 U/L but no RBCs on urinalysis. Which of the following symptoms would most likely also be present?

    • A.

      Arrhythmia

    • B.

      Hepatomegaly

    • C.

      Infl ammation of the metacarpophalangeal joints

    • D.

      Pain in a dermatomal distribution

    • E.

      Shuffling gait

    Correct Answer
    A. Arrhythmia
    Explanation
    45. The correct answer is A. The patient has experienced rhabdomyolysis secondary to extreme muscle strain. Rhabdomyolysis causes the release of muscle cell contents into the bloodstream, leading to an elevated creatine kinase level and myoglobinuria (red urine characterized by a urine dipstick test that is positive for blood but shows no RBCs on urinalysis). Release of intracellular potassium may lead to the development of significant arrhythmias and possibly death.
    Answer B is incorrect. Hepatomegaly is a nonspecific c sign of many medical conditions but is not a typical consequence of rhabdomyolysis.
    Answer C is incorrect. Inflammation of the metacarpophalangeal joints is characteristic of rheumatoid arthritis and is unrelated to rhabdomyolysis.
    Answer D is incorrect. Pain in a dermatomal distribution is characteristic of shingles and is unrelated to rhabdomyolysis.
    Answer E is incorrect. A shuffling gait may be seen in Parkinson’s disease and is unrelated to rhabdomyolysis.

    Rate this question:

  • 46. 

    Albinism is an autosomal recessive disorder in which a defi ciency in tyrosinase activity leads to a lack of pigmentation in the hair, skin, and eyes. The cells most involved in this condition are derived from which of the following germ cell layers?

    • A.

      Ectoderm

    • B.

      Endoderm

    • C.

      Mesoderm

    • D.

      Neuroectoderm, neural crest cells

    • E.

      Neuroectoderm, non-neural crest cells

    Correct Answer
    D. Neuroectoderm, neural crest cells
    Explanation
    46. The correct answer is D. The lack of pigmentation in the hair, skin, and eyes that is seen in albinism results from a lack of tyrosinase activity in melanocytes. Without tyrosinase, these cells are unable to convert tyrosine to melanin, putting the affected person at an increased risk of squamous and basal cell carcinoma (BCC) and malignant melanoma. Melanocytes, odontoblasts, pia and arachnoid mater, Schwann cells, cells of the ganglia, parafollicular C cells of the thyroid, chromaffin cells, the aorticopulmonary septum, pharyngeal arch skeletal components, and the neurocranium are all derived from neural crest cells, a specific c portion of the neuroectoderm.
    Answer A is incorrect. Although the ectoderm does give rise to the epidermis and the hair (as well as many other structures), it does not produce melanocytes.
    Answer B is incorrect. The endoderm is responsible for the epithelial lining of many internal organs as well as other structures, but plays no role in albinism.
    Answer C is incorrect. The mesoderm produces many structures, including the dermis, but does not produce melanocytes.
    Answer E is incorrect. The neuroectoderm cells outside of the neural crest are the source of the iris (as well as several other optic and non-optic structures) but do not give rise to melanocytes.

    Rate this question:

  • 47. 

    The father of a 7-year-old boy is contacted by his child’s schoolteacher because she is concerned about his inattentiveness during class. The teacher states that the boy appears to be daydreaming multiple times each day, during which time he blinks his eyes repeatedly. She reports that the boy’s daydreaming episodes are brief and he is able to refocus shortly following the daydream. What is the most appropriate therapy for the child’s underlying condition?

    • A.

      Carbamazepine

    • B.

      Clonazepam

    • C.

      Ethosuximide

    • D.

      Gabapentin

    • E.

      Methylphenidate

    • F.

      Tiagabine

    Correct Answer
    C. Ethosuximide
    Explanation
    47. The correct answer is C. The boy has a history consistent with absence seizures. On clinical examination, typical absence seizures appear as brief staring spells with no warning or postictal phase. Children are not responsive during the seizure and are amnestic of what happened during the attack. In fact, patients are generally unaware that a seizure has occurred. Classically, a regular and symmetric 3-Hz spike is found on electroencephalography. Ethosuximide is the primary treatment option in cases of absence (petit mal) seizures.
    Answer A is incorrect. Carbamazepine has been associated with the exacerbation of absence seizures.
    Answer B is incorrect. Clonazepam and the ketogenic or medium-chain triglyceride diet have been attempted to reduce seizure frequency. These adjunctive therapies, however, have limited efficacy.
    Answer D is incorrect. Gabapentin has been associated with the exacerbation of absence seizures.
    Answer E is incorrect. The teacher’s concerns regarding the boy are quite common in the case of absence seizures. Often such concerns will be incorrectly attributed to inattentiveness and may even lead to a misdiagnosis of attention deficit/hyperactivity disorder (ADHD). Methylphenidate, a central nervous system stimulant, is the cornerstone of therapy in ADHD.
    Answer F is incorrect. Tiagabine has been associated with the exacerbation of absence seizures.

    Rate this question:

  • 48. 

    A 53-year-old man presents to his physician with complaints of chest pain that worsens on exertion and at times of stress. An angiogram reveals fi brous plaques in the intima of the proximal portions of his coronary arteries. Fibrous plaques are indicative of which of the following pathologic processes?

    • A.

      Arteriosclerosis

    • B.

      Atherosclerosis

    • C.

      Medial calcifi c sclerosis

    • D.

      Mönckeberg’s arteriosclerosis

    • E.

      Takayasu’s arteritis

    Correct Answer
    B. Atherosclerosis
    Explanation
    48. The correct answer is B. Atherosclerosis begins with the formation of a fatty streak, which is caused by the deposition of cholesterol and cholesterol esters, lipid-laden macrophages (foam cells), calcium, and necrotic debris in the intima of the coronary arteries. Over time, the fatty streak becomes a proliferative plaque and then a complex atheroma under the influence of specific c risk factors such as smoking, hypertension, diabetes, hyperlipidemia, and family history. The most common arteries affected are the abdominal aorta, coronary, popliteal, carotid, renal, and mesenteric arteries.
    Answer A is incorrect. Arteriosclerosis is a general term for vascular disease characterized by rigidity caused by calcification and often thickening of blood vessels. The most common places for arteriosclerosis are the radial and ulnar arteries.
    Answer C is incorrect. Medial calcific sclerosis, also known as Monckeberg’s arteriosclerosis, involves the media of medium-sized muscular arteries.
    Answer D is incorrect. Monckeberg’s arteriosclerosis is characterized by ringlike calcifications that do not cause obstruction to arterial flow because the intima is not involved.
    Answer E is incorrect. Takayasu’s arteritis is also known as “pulseless disease” because of thickening of the aortic arch or proximal great vessels, which leads to weak pulses in the upper extremities and ocular disturbances.

    Rate this question:

Quiz Review Timeline +

Our quizzes are rigorously reviewed, monitored and continuously updated by our expert board to maintain accuracy, relevance, and timeliness.

  • Current Version
  • Mar 20, 2023
    Quiz Edited by
    ProProfs Editorial Team
  • Nov 19, 2012
    Quiz Created by
    Chachelly
Back to Top Back to top
Advertisement
×

Wait!
Here's an interesting quiz for you.

We have other quizzes matching your interest.